Understanding the Lagrangian and Deriving the Euler-Lagrange Equations

  • Thread starter gulsen
  • Start date
In summary, the conversation revolved around the formula \frac{\partial S}{\partial t} = -\mathcal{H}, which was being derived using the simplest/one dimensional Lagrangian. The conversation also touched upon the concept of functional derivatives and the notation used for differentials.
  • #1
gulsen
217
0
I'm trying to understand how [tex]\frac{\partial S}{\partial t} = -\mathcal{H}[/tex]. I put the simplest/one dimensional Lagrangian ([tex]mv^2/2-V[/tex]) and tried to derive it, but I failed:

[tex]\frac{\partial S}{\partial t} = \frac{\partial }{\partial t} \int_{t_i}^{t_f} Ldt[/tex]

noting that x and [tex]\dot{x}[/tex] is a function of t, expanding the integrand into power series and collecting only the first terms:

[tex]= \int_{t_i}^{t_f} \left( \frac{\partial (m\dot{x}^2)}{\partial \dot{x}}\frac{\partial \dot{x}}{\partial t} - \frac{\partial V}{\partial x} \frac{\partial x}{\partial t} + \frac{\partial L}{\partial t} \right) dt[/tex]

since [tex]-\frac{\partial V}{\partial x} = F[/tex] and because Lagrangian has no direct time dependence, the last term should drop. Also, [tex]\frac{\partial x}{\partial t} = \frac{dx}{dt}[/tex] since it's a function of t only (same goes for [tex]\dot{x}[/tex])

[tex]= \int_{\dot{x}(t_0)}^{\dot{x}(t_f)} m\dot{x} d\dot{x} + \int_{x(t_0)}^{x(t_f)} F dx[/tex]
which's not something like [tex]-\mathcal{H}[/tex]. Any ideas what has gone wrong? And could someone work out full derivation?
 
Last edited:
Physics news on Phys.org
  • #2
The dS/dt = - H applies when you consider S a function of the endpoint S(q_f, t_f), so it is actually [tex]\partial S/\partial t_f[/tex] if I remember well.
 
  • #3
S is a functional, not a function, it doesn't have partial derivatives, but functional derivatives, like Ga^teaux or Frechet...

So your [itex] \frac{\partial S}{\partial t} = - H [/itex] is what physicists call "bull****".

Daniel.
 
  • #4
dextercioby said:
S is a functional, not a function, it doesn't have partial derivatives, but functional derivatives, like Ga^teaux or Frechet...

So your [itex] \frac{\partial S}{\partial t} = - H [/itex] is what physicists call "bull****".

No, you should check Landau and Lif****z. At a certain moment (just before they introduce the minimum action principle of Maupertuis - I have the 3rd edition from Mir, in French), they consider the action, for a given initial point (q1,t1), as a function of the final point (q2,t2), using only the correct path:
[tex] S(q_1,t_1,q_2,t_2) = \int_{t_1}^{t_2} dt L(q(t),\dot{q}(t),t) [/tex] in which we already substituted the correct [tex]q(t)[/tex] which is the solution to the variational problem. As such, S is simply a function of the begin and end points, and by convention, we fix the begin point, which makes S then an ordinary function of the end points.
Turns out that [tex]\frac{\partial S}{\partial q_i}[/tex] equals [tex]p_i[/tex] (or minus ? I don't remember...) and [tex]\frac{\partial S}{\partial t}[/tex] equals [tex]-H[/tex]. The partial derivative is by changing the end time of the end point, without changing the point itself: this will slightly modify the dynamical solution (as now the trajectory between q_1 and q_2 will have to take a bit more time), and hence the action integral for that solution.
 
Last edited:
  • #5
There is a simple way to see how this comes about in all generality.
Consider the solution to the dynamical problem q(t), which is such that [tex]q_1 = q(t_1)[/tex] (point (1) ) and [tex]q_2 = q(t_2)[/tex] (point (2)). In other words, it is the extremal problem, solved:
[tex]
\delta \int_{t_1}^{t_2} dt L(q,\dot{q},t) = 0
[/tex]

The corresponding action corresponds to:
[tex]
S_1 = \int_{t_1}^{t_2} dt L(q(t),\dot{q}(t),t)
[/tex]

We can say that the point (q1,t1) and the point (q2,t2) are "on the path" q(t).

Now, consider that we lengthen the dynamical solution q(t) a bit further. In order to do so, there has to be continuity of the derivatives at point 2. We consider that we lengthen the path a bit to (q2 + dq, t + dt). We call this point, point (3). Note that we don't really have a choice for dq ! dq will be [tex]\dot{q} dt[/tex], because it is determined by the "initial" conditions of the arrival of the trajectory at point (2).

We can consider the dynamical problem between (1) and (3), but we know already that the path between (1) and (2) will be part of it. So our second action will be:
[tex]
S_2 = \int_{t_1}^{t_2+dt} dt L(q(t),\dot{q}(t),t) = S_1 + L(q_2,\dot{q}_2,t) dt
[/tex]

We can write this as:
[tex]
\frac{dS}{dt} = L
[/tex]

which reads: the derivative of the action (along the correct path) equals the lagrangian (at the end point we're considering).

But this is not what we are after. We wanted the solution of the dynamical problem, with t shifted from t2 to t2 + dt, but without changing the end point.

So we have to work back a bit. We can say that:
[tex]\frac{dS}{dt} dt = \frac{\partial S}{\partial q} dq + \frac{\partial S}{\partial t}dt [/tex]
from which we derive:

[tex]
\frac{\partial S}{\partial t} = \frac{dS}{dt} - \frac{\partial S}{\partial q} \frac{dq}{dt}
[/tex]

where we have to remember that the increments dq and dt were along a genuine dynamical path. If we now accept (can be shown too) that [tex]\frac{\partial S}{\partial q} = p[/tex], then we have:

[tex]
\frac{\partial S}{\partial t} = L - p \dot{q}
[/tex]

which is nothing else but - H.
 
Last edited:
  • #6
I see now, it's another matter than i had been thinking of in the first place.

A suggestion of notation would be to consider changing [itex] dt,dq [/itex] with [itex] \Delta t, \Delta q [/itex].

Keeping the d's means using differential forms where they shouldn't belong.

Daniel.
 
  • #7
dextercioby said:
A suggestion of notation would be to consider changing [itex] dt,dq [/itex] with [itex] \Delta t, \Delta q [/itex].

Keeping the d's means using differential forms where they shouldn't belong.

Yes, that's a good idea...
 
  • #8
Thanks vanesch, that leaves yo show [tex]\frac{\partial S}{\partial q} = p[/tex], but it appears to be as troublesome as the first question... (I'm a newbie to such things) I'll give it a try in the evening.

dextercioby said:
S is a functional, not a function, it doesn't have partial derivatives, but functional derivatives, like Ga^teaux or Frechet...

So your [itex] \frac{\partial S}{\partial t} = - H [/itex] is what physicists call "bull****".

Daniel.

This's simply a flame and I wonder where the admins are.

Daniel, thank you for your warm, enlightening comment. I'm glad that I don't fit into category of people whom you call physicist.
 
  • #9
Well, you can work out the variation of the action:
[tex] q \rightarrow q + \Delta q [/tex], as you do to derive the E-L equations in the first place, keeping the time boundaries fixed:

[tex] \Delta S = \left[\frac{\partial L}{\partial \dot{q}} \Delta q \right]_{t_1}^{t_2} + \int_{t_1}^{t_2}\left(
\frac{\partial L}{\partial q}- \frac{d}{dt}\frac{\partial L}{\partial \dot{q}}\right) \Delta q dt [/tex]

Usually, we say that the variation [tex] \Delta q [/tex] has to vanish on the end points, which makes the integrated term disappear. If otherwise Delta is a random small variation, then the only way to get this expression to 0, is by requiring the E-L equation to be valid. That's usually how the E-L equation is derived from the variational principle.

However, this time, we do things differently. We consider that the E-L equation IS satisfied (we are working with a solution to the dynamical problem), but we consider a variation which does NOT vanish at the end point (2) (although it does, at the end point (1)). We say that [tex] \Delta q (t_2) = \Delta q[/tex]
As such, the thing that remains is:
[tex] \Delta S = \frac{\partial L}{\partial \dot{q}} \Delta q \right]_{t_2} [/tex]
from which we can deduce that [tex]\partial S / \partial q_2 = p [/tex]
 
Last edited:
  • #10
gulsen said:
This's simply a flame and I wonder where the admins are.

Daniel, thank you for your warm, enlightening comment. I'm glad that I don't fit into category of people whom you call physicist.

Let's not get into a shouting contest here. Daniel misunderstood the original question, no need to get nervous over it.
 

1. What does the equation DS/dt = -H represent?

The equation DS/dt = -H represents the rate of change of a variable S with respect to time t, where the change is proportional to a constant H. This type of equation is known as a first-order differential equation.

2. How is the equation DS/dt = -H used in scientific research?

The equation DS/dt = -H is commonly used in physics, chemistry, and biology to model various physical and chemical processes. It is particularly useful in studying systems that experience continuous change over time, such as population growth, chemical reactions, and radioactive decay.

3. What is the significance of the constant H in the equation DS/dt = -H?

The constant H, also known as the rate constant, determines the speed at which the variable S changes over time. It is unique to each specific system and is dependent on various factors such as temperature, pressure, and concentration.

4. How is the equation DS/dt = -H solved?

The solution to the equation DS/dt = -H involves finding a function that satisfies the equation and the initial conditions of the system. This is typically done using mathematical techniques such as separation of variables, integrating factors, or using software programs for numerical solutions.

5. Are there any real-world applications of the equation DS/dt = -H?

Yes, there are many real-world applications of the equation DS/dt = -H. Some examples include modeling population growth and decline in biology, predicting the rate of decay of radioactive isotopes in chemistry, and studying the cooling of objects in physics. It is also commonly used in engineering and economics to understand and predict changes in various systems over time.

Similar threads

  • Advanced Physics Homework Help
Replies
2
Views
823
  • Advanced Physics Homework Help
Replies
11
Views
1K
  • Advanced Physics Homework Help
Replies
1
Views
984
  • Advanced Physics Homework Help
Replies
16
Views
912
  • Advanced Physics Homework Help
Replies
6
Views
299
  • Advanced Physics Homework Help
Replies
10
Views
572
  • Advanced Physics Homework Help
Replies
0
Views
124
  • Advanced Physics Homework Help
Replies
3
Views
1K
  • Advanced Physics Homework Help
Replies
1
Views
913
Replies
2
Views
966
Back
Top